Postać wykładnicza

Definicja. Postać wykładnicza i trygonometryczna. Zagadnienia związane z ciałem liczb zespolonych.
Speed094
Użytkownik
Użytkownik
Posty: 157
Rejestracja: 11 paź 2014, o 13:48
Płeć: Mężczyzna
Lokalizacja: Warsaw
Podziękował: 3 razy
Pomógł: 1 raz

Postać wykładnicza

Post autor: Speed094 »

Znaleźć
\(\displaystyle{ z=r\cdot e^{i\phi}\\ \frac{2|z|^2z}{z^3}=2i}\)

Czyli mam
\(\displaystyle{ \frac{2r^2\cdot r\cdot e^{i\phi}}{r^3\cdot e^{3i\phi}}=2i}\) ?
Gouranga
Użytkownik
Użytkownik
Posty: 1591
Rejestracja: 16 maja 2013, o 17:56
Płeć: Mężczyzna
Lokalizacja: Trójmiasto
Podziękował: 11 razy
Pomógł: 246 razy

Postać wykładnicza

Post autor: Gouranga »

Tak, teraz sporo można skrócić
Speed094
Użytkownik
Użytkownik
Posty: 157
Rejestracja: 11 paź 2014, o 13:48
Płeć: Mężczyzna
Lokalizacja: Warsaw
Podziękował: 3 razy
Pomógł: 1 raz

Postać wykładnicza

Post autor: Speed094 »

\(\displaystyle{ \frac{2r^2\cdot r\cdot e^{i\phi}}{r^3\cdot e^{3i\phi}}=2i\\
\frac{2e^{i\phi}}{e^{3i\phi}}=2i}\)
Gouranga
Użytkownik
Użytkownik
Posty: 1591
Rejestracja: 16 maja 2013, o 17:56
Płeć: Mężczyzna
Lokalizacja: Trójmiasto
Podziękował: 11 razy
Pomógł: 246 razy

Postać wykładnicza

Post autor: Gouranga »

mało

\(\displaystyle{ \frac{2e^{i\varphi}}{e^{3i\varphi}}=2i\\
\frac{1}{e^{2i\varphi}} = i\\
\frac{1}{e^{2i\varphi}} = e^{\frac{i\pi}{2}}\\
1 = e^{\frac{i\pi}{2}} \cdot e^{2i\varphi}\\
e^0 = e^{\frac{i\pi}{2} + 2i\varphi}\\
0 = {\frac{i\pi}{2} + 2i\varphi\\}\)


dasz radę już dalej obliczyć \(\displaystyle{ \varphi}\)?
Speed094
Użytkownik
Użytkownik
Posty: 157
Rejestracja: 11 paź 2014, o 13:48
Płeć: Mężczyzna
Lokalizacja: Warsaw
Podziękował: 3 razy
Pomógł: 1 raz

Postać wykładnicza

Post autor: Speed094 »

\(\displaystyle{ \frac{\pi}{4}}\)?
Gouranga
Użytkownik
Użytkownik
Posty: 1591
Rejestracja: 16 maja 2013, o 17:56
Płeć: Mężczyzna
Lokalizacja: Trójmiasto
Podziękował: 11 razy
Pomógł: 246 razy

Postać wykładnicza

Post autor: Gouranga »

\(\displaystyle{ -\frac{\pi}{4}}\) bo przenosisz jeden składnik na lewo i dzielisz przez \(\displaystyle{ 2i}\)

czyli rozwiązaniem są \(\displaystyle{ z: \quad z \in \CC \wedge Arg(z) = -\frac{\pi}{4}}\)
Speed094
Użytkownik
Użytkownik
Posty: 157
Rejestracja: 11 paź 2014, o 13:48
Płeć: Mężczyzna
Lokalizacja: Warsaw
Podziękował: 3 razy
Pomógł: 1 raz

Postać wykładnicza

Post autor: Speed094 »

No ale chyba jest gdzieś błąd bo arg jest w zakresieod \(\displaystyle{ 0}\) do \(\displaystyle{ 2 \pi}\)
Ostatnio zmieniony 12 gru 2014, o 08:10 przez yorgin, łącznie zmieniany 1 raz.
Powód: Poprawa wiadomości.
Gouranga
Użytkownik
Użytkownik
Posty: 1591
Rejestracja: 16 maja 2013, o 17:56
Płeć: Mężczyzna
Lokalizacja: Trójmiasto
Podziękował: 11 razy
Pomógł: 246 razy

Postać wykładnicza

Post autor: Gouranga »

Speed094, \(\displaystyle{ -\frac{\pi}{4} = \frac{7\pi}{4}}\), o kątach skierowanych słyszałeś?
Awatar użytkownika
yorgin
Użytkownik
Użytkownik
Posty: 12762
Rejestracja: 14 paź 2006, o 12:09
Płeć: Mężczyzna
Lokalizacja: Kraków
Podziękował: 17 razy
Pomógł: 3440 razy

Postać wykładnicza

Post autor: yorgin »

Gouranga pisze: czyli rozwiązaniem są \(\displaystyle{ z: \quad z \in \CC \wedge Arg(z) = -\frac{\pi}{4}}\)
Oraz \(\displaystyle{ z\neq 0}\).
Gouranga
Użytkownik
Użytkownik
Posty: 1591
Rejestracja: 16 maja 2013, o 17:56
Płeć: Mężczyzna
Lokalizacja: Trójmiasto
Podziękował: 11 razy
Pomógł: 246 razy

Postać wykładnicza

Post autor: Gouranga »

yorgin, istotnie, jak zwykle zapomniałem od początku wyznaczyć dziedzinę ...
ODPOWIEDZ